If apples cost $3.12 per pound and you have a bag that weighs 3pounds and 8 ounces, how much will you pay for those apples?

Answers

Answer 1

Answer: $10.92

Step-by-step explanation:

Since there are 16 ounces in a pound, 8 ounces would be equivalent to half a pound. Thus, 3 pounds and 8 ounces = 3.5 pounds.

Since apples cost $3.12 per pound and we have 3.5 pounds, the total cost would be $3.12*3.5 pounds = $10.92


Related Questions

if 8 out of 12 marbles in a bag are green, what is the probability that a marble selected at random from the bag will NOT be green?​

Answers

Answer:

The Probability that a marble selected at random from the bag will NOT be green is 1/3

Step-by-step explanation:

The probability of selecting a green bag

                                            = Total No.of green bags/ Total number of bags

                                           =8/12

                                          = 2/3

Then the probability not of selecting a green bag  

                                                = 1 - The probability of selecting a green bag

                                                 = 1-(2/3)

                                                = 1/3

2. Select all the equations that
describe the situation and then
find the solution.
Each notebook contains
60 sheets of paper. Andre
has 5 notebooks. How
many sheets of paper do
Andre's notebooks
contain?
i.y = 60÷5
ii. y = 5 times 60
iii. Y/5 = 60
iv. 5y = 60

Answers

Answer: i. y = 60÷5

Step-by-step explanation: If he has 60 sheets of paper and 5 notebooks, it would only make sense to divide the number of sheets by the number of notebooks to get the sum. Which is 12 btw! <33


URGENT Complete the table.
It is in the image below


Answers

Answer:

Step-by-step explanation:

Step-by-step explanation:

4=2×2

8=2×4

12=2×6

16=2×8

20=2×10

24=2×12

The frequency table represents the fraction of an hour
Rita spends writing in her journal each night. Represent
this data in the line plot.
16. What fraction of an hour did Rita spend most days
writing?
17. What is the difference between the greatest amount
of time spend writing and the least?

Answers

A line plot that represents the data is shown in the image attached below.

A fraction of an hour which Rita spend most days writing is 3/4.

The difference between the greatest amount of time spend writing and the least is 3/4.

What is a line plot?

In Mathematics and Statistics, a line plot is a type of graph that is used for the graphical representation of data set above a number line, while using crosses, dots, or any other mathematical symbol.

In this scenario and exercise, we would use an online graphing calculator to graphically represent the given data set on a line plot as shown in the image attached below.

Additionally, we can logically deduce that the mode of the data set is equal to 3/4 (the fraction of an hour Rita spend most days writing) because it has the highest frequency of 5.

Difference = greatest amount of time - least amount of time

Difference = 1 - 1/4

Difference = 3/4.

Read more on line plot here: brainly.com/question/28741427

#SPJ1

b) d=22,4 mm d) d= 7 km f) r=0,5 hm h) r= ² kr km 3 C≈ C≈ C≈ C≈​

Answers

The height is 230 feet after 9.125 seconds.

How do we calculate?

We have the  equation that describes the height of the object as a function of time as:

h(t) = -16t^2 + 145t + 2

We input the values and simplify:

-16t^2 + 145t + 2 = 230

-16t^2 + 145t - 228 = 0

we can use the quadratic formula, to solve this quadratic equation,

t = (-b ± √(b^2 - 4ac)) / 2a

where a = -16, b = 145, and c = -228.

t = (-145 ± √(145^2 - 4(-16)(-228))) / 2(-16)

t = (-145 ± √ (21025)) / (-32)

t = (-145 ± 145) / (-32)

t = 0.625 seconds or t = 9.125 seconds

In conclusion, the height is 230 feet after 9.125 seconds.

#complete question:

An object is thrown upward at a speed of 145 feet per second by a machine from a height of 2 feet off the ground. The height h of the object after t seconds can be found using the equation

When will the height be 230 feet?

Word problem down below: Fairly easy.

Answers

Based on the word problem, the answer is not one of the given options (E. NOTA).

How to explain the word problem

In this case, we have 9 letters and we want to select all of them, so n = 9 and r = 9. Therefore:

A = 9!

A = 362,880

B) The hypotenuse of a right triangle can be found using the Pythagorean theorem:

c^2 = a^2 + b^2

In this case, we have a = 9 and b = 12. Therefore:

c^2 = 9^2 + 12^2

c = 15

So, B = 15.

y - 9 = 3(x - 6)

y - 9 = 3x - 18

y = 3x - 9

The y-intercept of this line is -9, so C = -9.

Therefore, A + B + C = 362,880 + 15 - 9 = 362,886.

So the answer is not one of the given options (E. NOTA).

Learn more about word problem on

https://brainly.com/question/21405634

#SPJ1

At the school's holiday craft fair, Colette and her classmates get to make gingerbread houses. Their teacher divides a bag of licorice ropes evenly among the 9 students at the table. Each student gets 3 licorice ropes to decorate the roof of his or her gingerbread house.

Answers

Answer: So, what are you asking?

Step-by-step explanation:

Describe a real-world problem that can be modeled by a circle.

Answers

A circle-based model is highly suitable for the design and construction of roundabouts, which present themselves as practical solutions to tackle traffic-related issues.

Describe a real-world problem that can be modeled by a circle.

Roundabout intersections involve vehicles circulating around a central island in a counterclockwise direction. The circular form guarantees fluidity in traffic movements and significantly reduces the requirement for traffic lights and stops signs while enhancing security.

This model entails using a circle, ascertaining that the diameter corresponds with the width of the roadway, with its center accommodating the location of the island at equidistant distances from all contact points on the circle's periphery, thus ensuring optimal functionality.

Read more on circles here:https://brainly.com/question/24375372

#SPJ1

5. Jennifer solved a math problem. The answer. to her problem was in square inches. Which formula did Jennifer most likely use to solve her problem?
A P=(2xl) + (2xw)
B V=SxSxS
C P= 4xs
D A= SXS​

Answers

Answer:

D. A = s × s

Step-by-step explanation:

D. A = s × s is an area formula. Area is side times side. This is an area formula for a square. The units of area are square units. Since Jennifer has square inches in her answer, she was working on an area problem.

A. and C. are perimeter formulas. Perimeter is the distance around the outside of a shape. The units of perimeter are straight up plain, one-dimensional units like inches (or feet, meters, miles, km)

B. is a volume formula. The units of volume is cubic units or units to the third power.

The answer is D.

Is (3, 5) a solution to this system of equations?
y = 5
Y= -5/3x+10

yes
no

Answers

Yes, (3, 5) is a solution to this system of equations.

How do you calculate the equation system?

The system has a single solution if the m-values of the two equations differ. The system has no solution if the two equations have the same m-value but distinct b-values.

Substituting x = 3 and y = 5 into the equations, we get:

y = 5 (which is already given)

y = (-5/3)x + 10

y = (-5/3) * 3 + 10

y = -5 + 10

y = 5

Therefore, both equations are satisfied by the point (3, 5), making it a solution to the system of equations.

Yes, (3, 5) is a solution to this system of equations.

To know more about equations visit:-

brainly.com/question/10413253

#SPJ1

Question content area top
Part 1
For the statement​ below, write the claim as a mathematical statement. State the null and alternative hypotheses and identify which represents the claim.
An amusement park claims that the mean daily attendence at the park is people.

Answers

Mathematical statement  μ = 18000 where μ is the mean, Null hypothesis H₀ : μ = 18000, Alternate hypothesis Hₐ : μ ≠ 18000.

What does a mathematical statement mean to you ?

A mathematical statement is a meaningful string of words that can be either true or false.

For instance , 3+4=7 is always true. Consequently, it is a true statement.

Similar to this ,  3+4=8 is never true  Therefore,  it is a false statement.

Here the amusement park makes the claim that the mean daily attendence at the park is 18000 people

∴ μ = 18000 is the statement .

What is a null hypothesis ?

A null hypothesis is known as statistical hypothesis that proposes that no statistical significance exists in a set of given observations. Hypothesis testing is used to assess the credibility of a hypothesis by using sample data. Sometimes referred to simply as the "null," it is represented as H₀.

∴  Here the null hypothesis is μ = 18000

What do you understand by alternate hypothesis ?

The alternative hypothesis is an assertion used in statistical inference experiment. It is contradictory to the null hypothesis and indicated by Ha or H₁ . We can also say that it is simply an alternative to the null. In hypothesis testing, an alternative theory is a statement which a researcher is testing.

For instance :  A school principal claims that students in her school score an average of seven out of 10 in exams. The alternate hypothesis is μ ≠ 7.0.

Similarly here in this question alternate hypotesis will be μ ≠ 18000.

To know more about mathematical statement visit : https://brainly.com/question/17029275

#SPJ1

Substitute the supplied value and simplify both sides of the equation, if necessary. Then decide if the supplied value is or is not a solution.

m + 2(m + 1) = 14 {4}

Answers

The equation simplifies to 14 = 14, which is a true statement.

What is the purpose of the equation?

In mathematics, an equation  is defined as a set of numbers that contains operations and may contain a variable. Equations are used to solve for these unknown variables  using various operations such as addition, subtraction, multiplication and division.

Replacing m by 4, we get:

m + 2 (m + 1) = 14

4 + 2 (4 + 1) = 14

4 + 2(5) = 14

4 + 10 = 14

14 = 14

The equation simplifies to 14 = 14, which is a true statement. Therefore, the given value 4 is a solution to Eq.

Learn more about Equations here

https://brainly.com/question/29657992

#SPJ1

The diagram below not drawn to scale. Shows a triangle ABC which represent; the cross-section of a roof. BD is the perpendicular to ADC. Calculate the A) length= BD
B) measure of angle CBD
c) area of triangle ABC
d) bearings of B and C ​

Answers

Answer) : B measure of angle CBD

Step-by-step explanation: BD is perpendicular to ADC now we know that there is no scale so all you have to do is measure the length and we gat that answer.

The function = 50 − 1.6 can be used to determine the amount, in fluid ounces, of laundry detergent remaining after x loads of laundry have been done. What is the rate of change amount of laundry in fluid ounces with respect to the number of loads of laundry that have been done?

Answers

The rate of change of the amount of laundry with respect to the number of loads is a constant value of -1.6 fluid ounces per load.

What is linear equation?

Any equation that has terms that are either constants or the product of a constant and a variable of the first degree (exponent 1) is said to be linear. In other words, the equation's greatest degree for any variable is 1. One way to express a linear equation is as follows:

ax + b = 0

If an is not equal to zero, a and b are constants, and x is the variable. This is how a linear equation is typically expressed.

The given function is f(x) = 50 - 1.6x.

Now, the rate of change is the slope of the equation.

Comparing the equation with that of the linear equation y = mx + c we see that the slope is -1.6.

Hence, the rate of change of the amount of laundry with respect to the number of loads is a constant value of -1.6 fluid ounces per load.

Learn more about linear equation here:

https://brainly.com/question/11897796

#SPJ1

Can you prove this trigonometric identity?

tan1/2x(2cotx+tan1/2x)=1

Answers

The trigonometric identity has been proven as 1 in the space below

How to prove the identity

Given the identity:

tan(1/2x) (2cot(x) + tan(1/2x)) = 1

To prove this identity, let's start by expressing cot(x) in terms of tan(x):

cot(x) = 1 / tan(x)

Now, replace cot(x) in the given identity:

tan(1/2x) (2(1/tan(x)) + tan(1/2x)) = 1

Now, let's use the double angle formula for tan(x):

tan(x) = 2 * tan(1/2x) / (1 - tan^2(1/2x))

Replace tan(x) in the equation:

tan(1/2x) (2(1/(2 * tan(1/2x) / (1 - tan^2(1/2x))) + tan(1/2x)) = 1

Simplify the equation:

tan(1/2x) ((1 - tan^2(1/2x)) + tan(1/2x) * tan(1/2x)) = 1

Now, notice that the expression in the parentheses is equal to 1:

(1 - tan^2(1/2x)) + tan^2(1/2x) = 1

So, we can simplify the equation further:

tan(1/2x) * 1 = 1

This directly leads us to the original identity:

tan(1/2x) = 1

Thus, the trigonometric identity is proven.

Read more on trigonometric identities here: https://brainly.com/question/20094605

#SPJ1

Find the measure of angle A.
A
8x + 2
10x - 4
38°

Answers

Answer:

8x + 2 + 10x - 4 + 38 = 180

18x + 36 = 180

18x = 144, so x = 8

Angle A measures 8(8) + 2 = 64 + 2 = 66°.

What is m∠a?

A straight angle divided by a ray into two angles. The smaller angle has a measure of 50 degrees.
40°
50°
90°
130°

Answers

The answer is 130 degree
To find the answer look at the procedures
X+50 degree=180 degree
X=180 degree-50 degree
X=130 degree

The number of bacteria in a culture is n(t) after t hours. n(t)=500e0.15t How many bacteria are present after 12 hours. Round to the nearest whole number.

Answers

The requried,  3025 bacteria are present after 12 hours.

To find the number of bacteria present after 12 hours, we simply need to substitute t=12 in the given equation:

[tex]n(12) = 500e^{(0.15*12)}[/tex]

[tex]n(12) = 500e^{1.8}[/tex]

n(12) ≈ 3024.82

Rounding this to the nearest whole number, we get:

n(12) ≈ 47,416 ≈ 3025

Therefore, there are approximately 3025 bacteria present after 12 hours.

Learn more about half-life here:

https://brainly.com/question/24710827

#SPJ1

Find the range of the list of values. 3, 16, 22, 25, 24, 23, 16, 16

Answers

Answer:

range = 22

Step-by-step explanation:

the range is the difference between the largest and smallest values in the data set.

largest value = 25 , smallest value = 3

range = 25 - 3 = 22

Can anybody help me on the question.

ASAP

Answers

The hanger in the illustration will have the value x = 6, and it will remain balanced.

Define equations?

Mathematical expressions with two algebraic expressions on either side of the equals (=) sign are called equations. It demonstrates the equality of the expressions printed on the left and right sides. We have LHS = RHS (left hand side = right hand side) in each mathematical equation. To determine the value of an unknown variable that stands in for an unknown quantity, equations can be solved.

Here's the query,

Considering the graph,

The hanger's equation is as follows:

4x= 24

There are 4 numbers here that represent the value of the hanger.

So, x+x+x+x will be 24.

The equation is thus:

4x = 24.

Now, by resolving the equation, we can determine the value of x that balances the hanger:

4x = 24.

When you divide 4 on both sides:

x = 24/4

x = 6.

As a result, the hanger will remain in equilibrium for x = 6.

To know more about equations, visit:

brainly.com/question/29657983

#SPJ1

. Labor Variances Ellen Chenoweth, the manager of the city of St. Paul road maintenance shop, uses standards to judge performance. Because a clerk mistakenly discarded some labor records, however, Ellen has only partial data for April. She knows that the total direct-labor flexible-budget variance was $1,643 favorable. Moreover, a recent pay raise produced an unfavorable labor price variance for April of $1,157. The actual hours of input were 1,780 and the standard labor price was $20 per hour. 1. Find the actual labor rate per hour 2. Determine the standard hours allowed for the output achieved.

Answers

Labor Variances Ellen Chenoweth, the manager of the city of St. Paul road maintenance shop, uses standards to judge performance, the actual labor rate per hour for April was $19.35.

1. Actual Labor Rate per Hour:

The labor price variance formula is:

Labor Price Variance = (Actual Labor Rate - Standard Labor Rate) x Actual Hours

We are given that the labor price variance for April was unfavorable and equal to $1,157. We also know that the standard labor rate was $20 per hour. Using the formula above and plugging in the known values, we can solve for the actual labor rate:

-1,157 = (Actual Labor Rate - 20) x 1,780

-1,157 = 1,780Actual Labor Rate - 35,600

1,780Actual Labor Rate = 34,443

Actual Labor Rate = 19.35

Therefore, the actual labor rate per hour for April was $19.35.

2. Standard Hours Allowed for the Output Achieved:

The flexible-budget variance formula is:

Flexible-Budget Variance = (Actual Hours - Standard Hours) x Standard Labor Rate

We are given that the flexible-budget variance for April was favorable and equal to $1,643. We also know that the standard labor rate was $20 per hour.

Using the formula above and plugging in the known values, we can solve for the standard hours allowed:

1,643 = (1,780 - Standard Hours) x 20

1,643 = 35,600 - 20Standard Hours

20Standard Hours = 33,957

Standard Hours = 1,697.85

Therefore, the standard hours allowed for the output achieved in April was 1,697.85.

For more details regarding labor rate, visit:

https://brainly.com/question/28341634

#SPJ1

When Joann and Shana were counting the amount of money they had in their wallets, the total was at least$56 and at most $85. If Shana has only in her wallet, which of the following inequalities represents all possible values for the amount of money, j, that Joann has?

Answers

The inequality that represents all possible values for the amount of money that Joann has is 21 ≤ J ≤ 50.

What inequalities represents values for the amount of money that Joann has?

Let's call the amount of money that Joann has in her wallet "J" and the amount of money that Shana has in her wallet "S".

We know that the total amount of money they have is at least $56 and at most $85. Therefore, we can write two inequalities:

J + S ≥ 56 (the total is at least $56)

J + S ≤ 85 (the total is at most $85)

We also know that Shana has only $35 in her wallet. We can substitute this value for S in the inequalities to get an inequality that represents all possible values for the amount of money that Joann has:

J + $35 ≥ 56 (substituting S = $35 in the first inequality)

J + $35 ≤ 85 (substituting S = $35 in the second inequality)

Simplifying these inequalities by subtracting $35 from both sides, we get:

J ≥ 21 (the amount of money Joann has is at least $21)

J ≤ 50 (the amount of money Joann has is at most $50).

Read more about inequalities

brainly.com/question/25275758

#SPJ1

You need to cut the strongest beam out of a log with a diameter of 18 in. The strength of a wooden beam is directly proportional to the product of its width and the square of its height. What are the dimensions of the strongest beam? Find the exact value and then round your answer to the nearest hundredth.
Need answers ASAP

Answers

The dimensions of the strongest beam are 27 inches x 13.5 inches.

What is a dimension?

In general, dimension refers to a measurable extent of a physical quantity, such as length, width, height, depth, or time. These dimensions provide a framework for describing and measuring objects and events in the physical world. In geometry, dimension are refers to the number of coordinates needed to specify a point in a space. In physics, the concept of dimension is used to describe the properties of space and time.

We know that the strength of the beam is directly proportional to its width multiplied with the square of its height. Let's call the width of the beam "w" and the height "h". Then we can write the strength of the beam as:

S = kwh², where "k" is the constant of proportionality.

We want to find the dimensions of the strongest beam, which means we want to find the values of "w" and "h" that will maximize the strength "S". To do this, we need to find the maximum value of the function S = kwh² subject to the constraint that the diameter of the log is 18 inches.

The diameter of the log is equal to the width of the beam plus twice the height of the beam:

d = w + 2h

Since the diameter of the log is 18 inches, we can write:

w + 2h = 18

or

w = 18 - 2h

Substituting this expression for "w" into the equation for the strength of the beam, we get:

S = k × w(18-2h) × h²

Expanding and simplifying this expression, we get:

S = 36kh³ - 2kh⁴

To find the maximum value of S, we take the derivative of S with respect to h and set it equal to zero:

dS/dh = 108kh² - 8kh³ = 0

Simplifying this expression, we get:

h²(108 - 8h) = 0

This equation has two solutions: h = 0 and h = 13.5.

Since a beam with height equal to zero would have zero strength, we reject the solution h = 0. Therefore, the maximum strength is achieved when h = 13.5 inches.

To find the corresponding width, we can use the equation we derived earlier:

w = 18 - 2h = 18 - 27 = -9

Since the width of the beam cannot be negative, we reject this solution as well.

Therefore, the required dimensions of the strongest beam are:

Width = 2h = 2(13.5) = 27 inches

Height = h = 13.5 inches

To know more about dimensions visit:

brainly.com/question/9259034

#SPJ1

ABC is a right angled triangle. if B = 90°, AC = 96 cm, C = 30°. Find AB?

Answers

Answer:

We can use trigonometry to find AB.

First, we can use the fact that the sum of the angles in a triangle is 180° to find angle A:

A + B + C = 180°

A + 90° + 30° = 180°

A = 60°

Now, we can use the sine function to find AB:

sin A = opposite/hypotenuse

sin 60° = AB/96

AB = 96 * sin 60°

AB = 96 * √3/2

AB = 48√3

Therefore, AB is approximately 83.14 cm (rounded to two decimal places).

3. "Frank has four different credit cards, the balances and interest information of which are outlined in the table below.

He would like to consolidate his credit cards to a single credit card with an APR of 18% and pay off the balance in 24 months.

What will his monthly
credit card payment be?
Hint use the monthly payment formula where PV = (all balances combined), i=0.18/12, and n=24

A. $390.00

B. $462.91

C. $454.31

D. $52.00

Answers

Answer:

Therefore, Frank's monthly credit card payment will be $454.31.

Step-by-step explanation:

Monthly payment = (PV * r) / (1 - (1 + r)^-n)

PV = $2,500 + $1,000 + $3,000 + $1,500

= $8,000

r = 0.18 / 12

= 0.015

Monthly payment = ($8,000 * 0.015) / (1 - (1 + 0.015)^-24)

= $454.31

What is the equation in point-slope form of the line that passes through the points (7,5) and (−4,−1) ? Responses y+1=67(x+4) y plus 1 equals fraction 6 over 7 end fraction left parenthesis x plus 4 right parenthesis y+1=611(x+4) y plus 1 equals fraction 6 over 11 end fraction left parenthesis x plus 4 right parenthesis y+4=116(x+1) y plus 4 equals fraction 11 over 6 end fraction left parenthesis x plus 1 right parenthesis y−1=611(x−4)

Answers

The line that goes through the specified points has the equation into point-slope form is  [tex]y+1=6/11(x+4)[/tex].

What is a straight line equation?

X-axis points equal (7, -4).

Y-axis points are equal to (5, -1).

To determine the equation of the line passing through the specified points in point-slope form:

A straight line equation's direction and steepness are commonly described by its slope, which is also known as a line's gradient.

In mathematics, the following formula yields the slope of a line:

Slope(m) = [tex]y2-y1/x2-x1[/tex]

slope= [tex]\frac{-1-5}{-4-7}[/tex]

slope= 6/11

To find point slope:

[tex]y-y1=m(x-x1)\\y-(-1)=6/11(x-(-4)\\y+1=6/11(x+4)[/tex]

To know more about Slope visit:

https://brainly.com/question/19131126

#SPJ1

A checkout clerk at a department store is expected to complete 16 transactions every hour.
In the past 20 minutes, he completed 6 transactions.
Choose True or False for each statement.

Statements:

If the clerk continues at the same rate, he will meet his goal of 16 transactions in 1 hour.

At his current rate, the clerk will complete 12 transactions in 1 hour.

At his current rate, the clerk will complete 9 transactions every half hour.

If the clerk continues at this rate for a 6-hour shift, he will complete 96 transactions.

Answers

✓ - True If the clerk continues at the same rate, he will meet his goal of 16 transactions in 1 hour.

       ➜ This is true. The clerk is currently going at 6 transactions per 1/3 hours. 6 * 3 = 18, and 18 > 16.

✗ - False At his current rate, the clerk will complete 12 transactions in 1 hour.

       ➜  This is false. As stated above, he will complete 18.

✓ - True At his current rate, the clerk will complete 9 transactions every half hour.

       ➜ Let us set up a proportion to check. Then we will cross-multiply and solve.

[tex]\frac{20\;min}{6} =\frac{30\;min}{x}[/tex]

180 = 20x

x = 9

       ➜ Yes, he will complete 9 every half hour.

✗ - False If the clerk continues at this rate for a 6-hour shift, he will complete 96 transactions.

       ➜  We can utilize a proportion again. 6 hours is equal to 360 minutes. Then, we will cross-multiply and solve.

[tex]\frac{20\;min}{6} =\frac{360\;min}{x}[/tex]

20x = 2,120

x = 106

       ➜ He will complete 106 transactions, not 96. It depends on if they're asking for exactly 96 or at least 96.

A normal distribution has a mean of19 and a standard deviation of . What percent of values are from 19 to 34
what is the % of the values from 19to 34

Answers

It should be noted that 49.87% of the values fall between 19 and 34 in the original normal distribution.

How to calculate the percentage

For the lower bound of 19, the z-score is:

z = (19 - 19) / 5 = 0

For the upper bound of 34, the z-score is:

z = (34 - 19) / 5 = 3

Looking up the area under the curve for a z-score of 3 in a standard normal distribution table, we find it to be 0.9987. Looking up the area under the curve for a z-score of 0, we find it to be 0.5. Therefore, the area under the curve between z-scores of 0 and 3 is:

0.9987 - 0.5 = 0.4987 = 49.87%

Learn more about percentages on

https://brainly.com/question/24877689

#SPJ1

The___is the sum of the lengths of the sides of a closed figure. A-apothem B-area C-total D-perimeter

Answers

Answer: D-perimeter

Step-by-step explanation: The sum of the lengths of all the sides of a closed figure is called its perimeter.

Answer:

Perimeter

Step-by-step explanation:

The sum of the lengths of all the sides of a closed figure is called its perimeter.

Use slopes and y-intercepts to determine if the lines 15x−6y=−5 and 5x−2y=5 are parallel.

Answers

The two lines are parallel because they have thesame slope and different y intercept

How do we know parallel lines?

If two non-vertical lines that are in the same plane has the same slope. However the lines will have different y intercept.

A linear equation is an algebraic equation of the form y=mx+b. involving only a constant and a first-order (linear) term, where m is the slope and b is the y-intercept.

Therefore putting the equations 15x−6y=−5 and 5x−2y=5 into that form.

15x-6y = -5

-6y = -15x-5

y = 5x/2 +5/6

for the second equation,

5x-2y =5

-2y = -5x-5

y = 5x/2 +5/2

therefore , the two lines are parallel because they have the same slope( 5/2) and different y intercept.

learn more about parallel lines from

https://brainly.com/question/24607467

#SPJ1

Other Questions
The ligament bridging the larynx with the trachea is the ________ ligament. What constitutional crisis was revealed as a result of Shay's Rebellion? Chest compression will produce a ______ end-tidal CO2 How long does a complete depolarization-repolarization process (twitch contraction) last in a cardiac muscle cell? Paul, who works for a county sheriff's department, shot and killed Jake, stating that he believed Jake was a terrorist. However, Paul had no evidence for this belief. Jake's family may file a ____________ against Paul for denying Jake's right to life without due process. Padding underneath the torso when immobilizing an injured child is generally not necessary if he or she is:A. older than 8 to 10 years.B. complaining of severe back pain.C. immobilized on a long backboard.D. experiencing cardiopulmonary arrest. Solve for x trigonometry Write a numerical pattern that represents the rule y=x+0 starting with x=0 Newly licensed accident and health agents who intend to market Long-Term Care insurance must complete an (?)-hour course in Long-Term Care insurance in each of their first (?) years of licensing (two terms). You work for a lender that requires a 15% down payment and uses the standard debt-to-income ratio to determine a person's eligibility for a home loan. Of the following, choose the person that you would rate the highest on their eligibility for a home loan. Person APerson BPerson CPerson Dhome value$95,000$107,000$120,000$128,000income$46,000$53,000$58,000$60,000savings$20,000$13,910$18,000$19,200recurring debt$310$198$265$400a. Person Ab. Person Bc. Person Cd. Person D PQRS are points on a circle with the centre O PS is a diameter of the circle Angle PQR=136 Work out the size of angle RPS How does decreasing quantum affect wait time? Question 40Following a disaster raw food should bea. washed in bleachb. cooked wellc. avoided if possibled. both b and c The Global Compact: A. is a code of conduct.B. enforces its principles.C. advances its principles as an "aspirational" set of "shared values."D. requires mandatory participation. Researchers believe that most newborn reflexes disapear during the first six months due to a gradual increase in voluntary control over behavior as the ______develops The nurse is reviewing the laboratory reports of four patients. Which patient's test results indicates possible type 2 diabetes mellitus?a. Patient A; Fasting = 98 mg/dL; Random = 150 mg/dLb. Patient B; Fasting = 105 mg/dL; Random= 175 mg/dLc. Patient C; Fasting = 112 mg/dL; Random = 195 mg/dLd. Patient D; Fasting = 135 mg/dL; Random = 230 mg/dL macy's buys white, pinpoint oxford blouses at $14 each and sells them at $30 each. macy's percentage (of cost) markup is ________ percent. a. 46.7 b. 87.5 c. 53.3 d. 114.3 Suppose output per worker is y=10k^(.05) , the saving rate is 20%, and the depreciation rate is 5%. If the amount of capital per worker is currently 10, what will it be next period?Select an answer and submit. For keyboard navigation, use the up/down arrow keys to select an answer.a. 10.9b. 13.4c. 15.8d. 17.3 Where are earthquakes and volcanoes most commonly found?a) in the higher latitudesb) at the edges of oceansc) along tectonic plate boundariesd) around the equatore) the location is random When J. J. Thomson discovered the electron, what physical property of the electron didhe measure?A) its charge, e D) its mass, mB) its charge-to-mass ratio, e/m E) its atomic number, ZC) its temperature, T